You are on page 1of 7
INDETERMINATE FORMS Introduction Let f(x) and g(x) be two functions defined on an interval 1 " fy Af Him £60 and tim 80 both exist and tiny 8g Othen fim’ exists. We also know that if lim f(x) as x —> ais finite and non zero then, even if lim g (x) = 0, the limit of the ratio f (x)/g(x) as x a exists and is equal to zero, But what will happen if lim f (x) as.v a is infinite or ze10? In that case this ratio is called indeterminate form. Precisely, we may define it as follows: fle) 0 Definition: The ratio“ .(,) is said to represent the indeterminate form 5 as x a if Jim f(x) =lim g(x) =0 «0 os types of indeterminate forms like 00 S—.°—=5. "= " etc. 0 L Hopital’s Rule for ¢ form Statement: Let f(x) and g(x) be two functions defined on an interval I such that flim) =timgcry=0 i) f'(x) and g'(x) exist and g (x) + Oand g'(x) Oforall xe [a- 5,a+ 3], 5 > Oercept possibly at.x= a fo Gi) im then tim him £ Ht g(x) 0g L’ Hopi Yim yt s Rule for Statement: Let (x) and g(x) be two functions defined on an interval I such that @ Tims (x) = lim fx) =e (ii) JS’) and g* (x) exist and g (x) 4 Oand g(x) 4 Oforallx ela— 5.a+ 5), 5 > O except possibly at =a Gi) then Examples Solution: Let us write f (x) = cos wand g(x) = cos O= 1— Now f (0) = =Oand g(0) =0 0 FO) tiny a iS of 5 form. Thus, using theorem 3.1, we have =H g(x) ie. But as x — 0, sin.x = 0 and 2x i.e. we have, again, got a 0/0 form. So, using remark 1, we have tim LOSS jim SOX _ £080 1 lige Ha 9 Examples A student has evaluated the limit as follows: xit3x-4 tim ———— ml 3b lim m1 — Set desl omg 2 Do you agree with this answer? Give reasons. Solution: No, we don’t agree with the solution done because 3 3x7 lim 0 lim [Fis not of 5 form, rather its value is ~ 3 therefore, the further application of L? Hopital’s rule is not required Remark 3: As is evident from the example 3, we should check at each step, before using the L’ Hopital’s o Rule that the limit ofthe expression is ofthe form 7 , otherwise, you may get misleading results. Examples Evaluate (ar Solution: Since tan (= |= and tan so we have, by theorem 3.2, _ tan3x (2 | lim“ “(= tom nf lan (co J . 3see73x (0, n = = form x 0 ) 6sin.xcos.x Il Tésinaxcos3x cecesteseestenees “ (L) We know that 2 sin A cos A= sin 2A, therefore, (I) is tim 2225 | 8 form oon) so Sin6x Example — What should be the values of a and b if, 0 Solution: Since expression on the left hand side is of > form, so using the theorem 3.1, x(I—acosx)+bsinx we have tim 3 A.0OS.K+X(ASIN-X) +b COS By For the further application of the L’ Hopital’s rule we should have limit of numerator as well as of denominator to be zero separately. Thus we have the first equati 1a cos +0 (asin0) +b cos0=0 ie loa b=0 Assuming (1) to be true, we can apply L’ Hopital rule again, Hence we have nx +-x(acosx)—bsinx = lim extanon sy ot Or axcos.x+ Qa-b)sin x/ 0 m | 5 form | 0 or \ 4 cos.xansin x +(2a~b) cos. ji osza eran Cabos 6 “Thus we have got another relation as By the statement of the problem, the limit of (II) is, 4 €080—0 (a sin) +Qa—b)coss_1 6 ie, 3a-b eID) 1 Ll Solving (1) and (III), we geta = 5 and b=— > Indeterminate forms of the type 0 x oo Rule: if lim (()=0 and mg(x)= then lim[ f(x) g(x)] is said to be of the form 0 x oo xa xa xa In this case we rewrite the functions as FO gQo= T or 2 Boy fF) so that [ f (x) g(x)] takes 0 © the form OTS BS and hence can be evaluated using L’ Hopital’s Rules. Example Show that lim /()=0 (1 ~ cos x) cot. x=0 Solution: lim (1—cos.x) cot (is 0.x 2> form, so by above rule) x = 0 = iy S259 (9 form} soe tan tim 2 in 0 m0 se Indeterminate forms of the type oo — co If lim_ lim f(0)=lim g@)= then lim [f()—80)] is said to be of the form c — co To evaluate such limits, we have % 1 Example: Evaluate lim[ cot" x-— = oo and lim 0 Solution: As lim cor”. We rewrite the expression to be evaluated as mX + 608 21)~ (Cos 2x) a posta (0 form] 2x* 0 sin 2v-+.x+.XeO8 2: 4 “ 6 Indeterminate forms of the type 17 20° Example; Evaluate lim (cos.x)!" Solution: It is of the form |= Let _y=(cos.x)"" => lim log 3 m0 = tim SS" *0 2cosx—2xsinx cose oO, . ( form } 0 => logy=-% >yse* Question Why L’ Hopital’s rule does not apply to the problem ? col Solution: We can rewrite the given expression as 1) ele) mG si(2) tim —_\*/ because lim ((sin x)/x ao 1 = li nasi ) mp : 1) (4) does not exist for x= 0, we can’t find out the limit of the numerator and denominator 0 separately to get a 7 form. Therefore L’Hopital’s rule is not applicable.

You might also like